Đến nội dung

Fr13nd nội dung

Có 76 mục bởi Fr13nd (Tìm giới hạn từ 25-04-2020)



Sắp theo                Sắp xếp  

#656583 Đề thi chọn đội tuyển Quốc gia tỉnh Thanh Hóa năm 2016-2017 ngày 1

Đã gửi bởi Fr13nd on 03-10-2016 - 19:00 trong Thi HSG cấp Tỉnh, Thành phố. Olympic 30-4. Đề thi và kiểm tra đội tuyển các cấp.

ai có đáp án đăng hộ mình với, cảm ơn.




#656511 Đề thi chọn đội tuyển Quốc gia tỉnh Thanh Hóa năm 2016-2017 ngày 1

Đã gửi bởi Fr13nd on 02-10-2016 - 22:01 trong Thi HSG cấp Tỉnh, Thành phố. Olympic 30-4. Đề thi và kiểm tra đội tuyển các cấp.

bạn nào giải được bài 2 không, giúp mình với 




#646761 Cho ba số thực dương a,b,c. Chứng minh rằng:

Đã gửi bởi Fr13nd on 27-07-2016 - 17:07 trong Bất đẳng thức và cực trị

mỗi cái cộng 1/2 là xong 




#640390 Chứng OQ vuông góc QP

Đã gửi bởi Fr13nd on 14-06-2016 - 22:54 trong Hình học phẳng

Tứ giác ABCD nội tiếp (O), AC cắt BD tại P, (ABP) cắt (CDP) tại điểm thứ 2 là Q, chứng minh OQ vuông QP. Xin cảm ơn




#640344 Tuần 3 tháng 6/2016: Bài toán xuyên tâm

Đã gửi bởi Fr13nd on 14-06-2016 - 20:52 trong Chuyên mục Mỗi tuần một bài toán Hình học

Tranh thủ đêm khuya:
Lời giải:
$BE, CF$ thứ tự cắt lại $(K)$ tại $X, Y$.
Ta có $\widehat{EXF}=\widehat{EDF}=\widehat{BAC}$(do $AEDF$ là hình bình hành) $=\widehat{BNF}$. Do đó $X, B, N, F$ đồng viên. Tương tự thì $Y, C, M, E$ đồng viên.
Đặt $(S)=(CMEY)$, $(R)=(BNFX)$. Theo giả thiết thì ta có: $O, S, P$ thẳng hàng và $O, R, Q$ thẳng hàng.
Ta có $\widehat{ESC}=2\widehat{EMC}=2\widehat{BAC}=\widehat{BOC}$, mặt khác $\triangle ESC, \triangle BOC$ thứ tự cân tại $S, O$ nên $\triangle ESC \sim \triangle BOC$, suy ra $\triangle CSO \sim \triangle CEB$.$(1)$
Ta lại có $\widehat{MSE}=2\widehat{ACM}=\widehat{MPC}$(do $(P;PM)$ tiếp xúc $AC$). Do $\triangle MSE, \triangle MPC$ thứ tự cân tại $S, P$ nên $\triangle MSE \sim \triangle MPC$, do đó $\triangle MSP \sim \triangle MEC$. $(2)$
Sử dụng hai cặp tam giác đồng dạng ở $(1)$ và $(2)$ ta có $\frac{SP}{SO} = \frac{SP}{SM}.\frac{SC}{SO}=\frac{EC}{EM}.\frac{EC}{EB}=\frac{EB}{EA}.\frac{EC}{EB}=\frac{EC}{EA}$. Tương tự thì $\frac{RO}{RQ}=\frac{FA}{FB}$. Mặt khác, $AEDF$ là hình bình hành, nên $\frac{EC}{EA}=\frac{DC}{DB}=\frac{FA}{FB}$. Do đó $\frac{SP}{SO}=\frac{RO}{RQ}$.
Đường thẳng qua $R$ song song $OP$ cắt $PQ$ tại $K$. Theo Thales thì $\frac{SP}{SO}=\frac{RO}{RQ}=\frac{KP}{KQ}$, do đó $KS \parallel OQ$. Dễ thấy $K$ thuộc trung trực $EY, FX$ nên $K$ là tâm ngoại tiếp $\triangle DEF$. Ta có đpcm.
P/s: bài này của thầy hay quá :D

em muốn hỏi anh sao anh xác định được 2 tứ giác nội tiếp ở đầu ạ để lấy tâm nữa ạ, bài giải anh hay quá




#634203 Topic yêu cầu tài liệu THPT

Đã gửi bởi Fr13nd on 20-05-2016 - 02:34 trong Tài liệu tham khảo khác

Ai có tài liệu về các bài dãy số, giới hạn có dạng như trong các kì thi VMO, TST cho mình xin, cảm ơn.




#632196 Tuần $2$ tháng $5/2016$: Bài toán phân giác liên quan đến...

Đã gửi bởi Fr13nd on 10-05-2016 - 01:52 trong Chuyên mục Mỗi tuần một bài toán Hình học

AP, BC, đường nối 2 chân đường phân giác từ E,F bài cho đồng quy đúng không ạ, em còn phần đó, tưởng dễ mà khó quá :v




#630905 Tìm a,b,c nguyên dương: $(a^3+b).(a+b^3) = 2^c$

Đã gửi bởi Fr13nd on 02-05-2016 - 22:05 trong Số học

Giải như sau:

 

TH1: $a=b$ dễ thấy $a=b=1$

TH2: Giả sử $a>b$. Từ điều kiện đề bài đặt $a^3+b=2^m,a+b^3=2^n$. Có $a^3+b>a+b^3$ nên $m>n\Rightarrow a+b^3|a^3+b$ $(1)$

Mặt khác, gọi $d=\gcd(a,b)\Rightarrow a=dx,b=dy$ với $(x,y)=1$ $\Rightarrow d^2(y+d^2x^3)(x+d^2y^3)=2^c$

Nếu $d\neq 1$ thì $d$ chẵn. Mà $y+d^2x^3,x+d^2y^3$ cũng chẵn nên $x,y$ chẵn ( vô lý). Do đó $(a,b)=1$

$\Rightarrow (1)\Leftrightarrow 2^n =a+b^3|b^8-1=(b^4-1)(b^4+1)$ suy ra $b$ lẻ.

Mà dễ thấy $\gcd(b^4-1,b^4+1)=2$ nên hoặc $2^{n-1}|b^4-1$ hoặc $2^{n-1}|b^4+1$

+) Nếu $2^{n-1}|b^4+1$: Thấy rằng vì $b$ lẻ nên $v_2(b^4+1)=1$, do đó $n-1=0,1\rightarrow n=1,2$. Thử thấy vô lý

+) Nếu $2^{n-1}|b^4-1=(b^2-1)(b^2+1)$. Tương tự như trên: $\gcd(b^2-1,b^2+1)=2$ nên hoặc $2^{n-2}|b^2+1$ hoặc $2^{n-2}|b^2-1$

Trường hợp $2^{n-2}|b^2+1$ mà $2||b^2+1$ nên $n-2=0,1$. Thử ta thấy không thỏa mãn

Trường hợp $2^{n-2}|b^2-1$. Đặt $b^2=2^{n-2}t+1$. Nếu $t=1\Rightarrow (b-1)(b+1)=2^{n-2}$. Phương trình tích ta dễ dàng tìm được $b=3$. Từ đó có bộ $(a,b,c)=(5,3,12)$ và hoán vị $(a,b)$. Nếu $t\geq 2$ thì $2^n=a+b^3>b(b^2+1)\geq b(2^{n-1}+2)>2^{n-1}b\Rightarrow b<2\rightarrow b=1\rightarrow 2^{n-2}+1=1$ (vô lý)

Vậy $(a,b,n)=(1,1,2), (5,3,12),(3,5,12)$

ký hiệu kia là số mũ phải không bạn, bạn có tài liệu, một số bài tập nào đề cập đến số mũ không, thanks.




#630900 Tìm a,b,c nguyên dương: $(a^3+b).(a+b^3) = 2^c$

Đã gửi bởi Fr13nd on 02-05-2016 - 21:56 trong Số học

@nuoccam: Ơ mình giải thích cụ thể thế còn gì !? Chính là thêm bớt như bạn Fr13nd nói đấy

@Fr13nd: Trong phương trình $d^2(y+d^2x^3)(x+d^2y^3)=2^c$ thì hiển nhiên $d$ là ước của $2^c$. Nếu $d$ lẻ thì $d=1$, nếu $d\neq 1$ thì hiển nhiên $d$ chẵn và có dạng $2^t$ nào đó với $t\leq c$

xl, không để ý đoạn đó :v




#630899 Tìm a,b,c nguyên dương: $(a^3+b).(a+b^3) = 2^c$

Đã gửi bởi Fr13nd on 02-05-2016 - 21:55 trong Số học

Ta có tính chất sau: nếu a.b=$2^c$ thì (a=$2^m$ và b=$2^n$) (m+n=c) a,b,c,m,n là STN

tính chất cơ bản mà bạn  :icon6:

tính chất này mình biết rồi nhưng bạn viết sai 




#630897 Tìm a,b,c nguyên dương: $(a^3+b).(a+b^3) = 2^c$

Đã gửi bởi Fr13nd on 02-05-2016 - 21:53 trong Số học

??? a=2^m thì a là snt ???




#630894 Tìm a,b,c nguyên dương: $(a^3+b).(a+b^3) = 2^c$

Đã gửi bởi Fr13nd on 02-05-2016 - 21:50 trong Số học

đoạn này thế này mà chị làm tắt làm e chẳng hiểu gì  :angry:  :D

thêm bớt thôi




#630893 Tìm a,b,c nguyên dương: $(a^3+b).(a+b^3) = 2^c$

Đã gửi bởi Fr13nd on 02-05-2016 - 21:49 trong Số học

chưa hiểu ý bạn




#630886 Tìm a,b,c nguyên dương: $(a^3+b).(a+b^3) = 2^c$

Đã gửi bởi Fr13nd on 02-05-2016 - 21:42 trong Số học

Giải như sau:

 

TH1: $a=b$ dễ thấy $a=b=1$

TH2: Giả sử $a>b$. Từ điều kiện đề bài đặt $a^3+b=2^m,a+b^3=2^n$. Có $a^3+b>a+b^3$ nên $m>n\Rightarrow a+b^3|a^3+b$ $(1)$

Mặt khác, gọi $d=\gcd(a,b)\Rightarrow a=dx,b=dy$ với $(x,y)=1$ $\Rightarrow d^2(y+d^2x^3)(x+d^2y^3)=2^c$

Nếu $d\neq 1$ thì $d$ chẵn. Mà $y+d^2y^3,x+d^2y^3$ cũng chẵn nên $x,y$ chẵn ( vô lý). Do đó $(a,b)=1$

$\Rightarrow (1)\Leftrightarrow 2^n =a+b^3|b^8-1=(b^4-1)(b^4+1)$ suy ra $b$ lẻ.

Mà dễ thấy $\gcd(b^4-1,b^4+1)=2$ nên hoặc $2^{n-1}|b^4-1$ hoặc $2^{n-1}|b^4+1$

+) Nếu $2^{n-1}|b^4+1$: Thấy rằng vì $b$ lẻ nên $v_2(b^4+1)=1$, do đó $n-1=0,1\rightarrow n=1,2$. Thử thấy vô lý

+) Nếu $2^{n-1}|b^4-1=(b^2-1)(b^2+1)$. Tương tự như trên: $\gcd(b^2-1,b^2+1)=2$ nên hoặc $2^{n-2}|b^2+1$ hoặc $2^{n-2}|b^2-1$

Trường hợp $2^{n-2}|b^2+1$ mà $2||b^2+1$ nên $n-2=0,1$. Thử ta thấy không thỏa mãn

Trường hợp $2^{n-2}|b^2-1$. Đặt $b^2=2^{n-2}t+1$. Nếu $t=1\Rightarrow (b-1)(b+1)=2^{n-2}$. Phương trình tích ta dễ dàng tìm được $b=3$. Từ đó có bộ $(a,b,c)=(5,3,12)$ và hoán vị $(a,b)$. Nếu $t\geq 2$ thì $2^n=a+b^3>b(b^2+1)\geq b(2^{n-1}+2)>2^{n-1}b\Rightarrow b<2\rightarrow b=1\rightarrow 2^{n-2}+1=1$ (vô lý)

Vậy $(a,b,n)=(1,1,2), (5,3,12),(3,5,12)$ 

 

không hiểu đoạn kia ai giải thích hộ đc k 




#627022 CMR: tồn tại $N$ số ng dương liên tiếp trong đó có đúng 2015 số dạn...

Đã gửi bởi Fr13nd on 14-04-2016 - 01:27 trong Số học

N nguyên dương




#627021 Chứng minh rằng $v_{p}(\frac{x^{k} + y^...

Đã gửi bởi Fr13nd on 14-04-2016 - 01:23 trong Số học

Dựa vào LTE ta chỉ cần chứng minh nếu $x^k+y^k \vdots p$ thì $x+y \vdots p$

Thật vậy, do $(k,p-1)=1$ nên là $k$ lẻ và tồn tại $u,v$ ko âm sao cho $uk-v(p-1)=1$

Ta có : $x^{k}\equiv -y^k\equiv (-y)^{k}(modp)\Rightarrow x^{uk} \equiv(-y)^{uk}(modp)$

và do $(x,p)=(y,p)=1$ : $x^{p-1}\equiv 1\equiv (-y)^{p-1}(modp)\Rightarrow x^{v(p-1)} \equiv(-y)^{v(p-1)}(modp)$

suy ra $x \equiv -y(modp)$ $\Rightarrow x+y \vdots p$ $đpcm$

đoạn đó bạn hạ bậc kiểu gì vậy 




#623110 Xin tài liệu hình học phẳng

Đã gửi bởi Fr13nd on 27-03-2016 - 23:18 trong Hình học phẳng

Ai cho mình xin tài liệu về một số bổ để quen thuộc của hình học phẳng




#620313 Chuyên đề số học của diễn đàn VMF

Đã gửi bởi Fr13nd on 14-03-2016 - 22:38 trong Tài nguyên Olympic toán

ấn phẩm sai nhiều quá, điển hình là bước phân tích nhân tử trang 63, ví dụ 4.8 




#616027 Thắc mắc về Bất đẳng thức

Đã gửi bởi Fr13nd on 20-02-2016 - 01:40 trong Bất đẳng thức và cực trị

thường được sử dụng các bài biến đổi tương đương, dồn biến,... thôi 




#616026 Một số câu Bất đẳng thức và Tìm GTLN, GTNN trong đề thi thử đại học

Đã gửi bởi Fr13nd on 20-02-2016 - 01:38 trong Bất đẳng thức và cực trị

 

p/s: câu 3 cứ \pi thôi, nó ra chứ $\pi$ đẹp đẽ kiêu sa, cứ làm cái dấu $\Pi$ làm cái gì vậy, xấu bm ra

đọc quả này mà đéo nhịn nổi cười, tìm mãi mới thấy 1 bựa nhân :))




#612350 $\frac{2a^{2}-bc}{b^{2}+c^{...

Đã gửi bởi Fr13nd on 01-02-2016 - 21:54 trong Bất đẳng thức và cực trị

Đoạn này có vấn đề với $2a^{2} \gegslant b^{2}+c^{2}$

Bài này có thể giải được bằng phép tương đương,nhưng hơi dài dòng nên xin không viết ra  :closedeyes:

quy đồng rồi chứng minh chay á :))




#612348 $\frac{2a^{2}-bc}{b^{2}+c^{...

Đã gửi bởi Fr13nd on 01-02-2016 - 21:53 trong Bất đẳng thức và cực trị

Ta có $\sum \frac{2a^2-bc}{b^2+c^2-bc}=\sum (1-\frac{b^2+c^2-2a^2}{b^2+c^2-bc})\geq \sum (1-\frac{b^2+c^2-2a^2}{bc})=3-\sum \frac{b^2+c^2-2a^2}{bc}=3-\frac{\sum ab(a+b)-2(a^3+b^3+c^3)}{abc}$

Mà $2(a^3+b^3+c^3)\geq a^3+b^3+c^3+3abc\geq \sum ab(a+b)(bdt- Shur)$

=>$\sum \frac{b^2+c^2-2a^2}{bc}\leq 0=> VT\geq 3$

đoạn này nếu tử mà âm thì coi như là hỏng rồi bạn, khắc phục được không  :mellow:




#612333 $\frac{2a^{2}-bc}{b^{2}+c^{...

Đã gửi bởi Fr13nd on 01-02-2016 - 21:13 trong Bất đẳng thức và cực trị

Ta có $\sum \frac{2a^2-bc}{b^2+c^2-bc}=\sum (1-\frac{b^2+c^2-2a^2}{b^2+c^2-bc})\geq \sum (1-\frac{b^2+c^2-2a^2}{bc})=3-\sum \frac{b^2+c^2-2a^2}{bc}=3-\frac{\sum ab(a+b)-2(a^3+b^3+c^3)}{abc}$

Mà $2(a^3+b^3+c^3)\geq a^3+b^3+c^3+3abc\geq \sum ab(a+b)(bdt- Shur)$

=>$\sum \frac{b^2+c^2-2a^2}{bc}\leq 0=> VT\geq 3$

mình đang cố đi theo hướng cauchy schwartz, bạn có ý tưởng gì không, cảm ơn.




#611835 $\frac{2a^{2}-bc}{b^{2}+c^{...

Đã gửi bởi Fr13nd on 30-01-2016 - 21:53 trong Bất đẳng thức và cực trị

 cho a,b,c>0 chứng minh, $\frac{2a^{2}-bc}{b^{2}+c^{2}-bc}+\frac{2b^{2}-ac}{a^{2}+c^{2}-ac}+\frac{2c^{2}-ab}{a^{2}+b^{2}-ab}\geqslant 3$




#611714 $F_{10^{k}}$ $(k{\geq}1)$ luôn tận cùng bằng 5

Đã gửi bởi Fr13nd on 29-01-2016 - 22:39 trong Số học

Dễ thấy đề sai vì giữa 0 và 1 không co nguyên tố

cần bắt bẻ vậy không bạn, nếu sửa đề đúng bằng cách thêm điều kiện thì bạn có làm được không :))